This polynomial limit doesn't exist but I can't seem to prove it.

The name of the pictureThe name of the pictureThe name of the pictureClash Royale CLAN TAG#URR8PPP











up vote
1
down vote

favorite












$$lim_(x,y)to(0,0)fracx^2+y^2+5xyx-y$$



I've tried approximating using parabolas and even use polar coordinates but it seems to always result in $0$.
The limit seems simple so I must be forgetting something basic.
Hints? Thank you.










share|cite|improve this question









New contributor




João Simões is a new contributor to this site. Take care in asking for clarification, commenting, and answering.
Check out our Code of Conduct.























    up vote
    1
    down vote

    favorite












    $$lim_(x,y)to(0,0)fracx^2+y^2+5xyx-y$$



    I've tried approximating using parabolas and even use polar coordinates but it seems to always result in $0$.
    The limit seems simple so I must be forgetting something basic.
    Hints? Thank you.










    share|cite|improve this question









    New contributor




    João Simões is a new contributor to this site. Take care in asking for clarification, commenting, and answering.
    Check out our Code of Conduct.





















      up vote
      1
      down vote

      favorite









      up vote
      1
      down vote

      favorite











      $$lim_(x,y)to(0,0)fracx^2+y^2+5xyx-y$$



      I've tried approximating using parabolas and even use polar coordinates but it seems to always result in $0$.
      The limit seems simple so I must be forgetting something basic.
      Hints? Thank you.










      share|cite|improve this question









      New contributor




      João Simões is a new contributor to this site. Take care in asking for clarification, commenting, and answering.
      Check out our Code of Conduct.











      $$lim_(x,y)to(0,0)fracx^2+y^2+5xyx-y$$



      I've tried approximating using parabolas and even use polar coordinates but it seems to always result in $0$.
      The limit seems simple so I must be forgetting something basic.
      Hints? Thank you.







      limits






      share|cite|improve this question









      New contributor




      João Simões is a new contributor to this site. Take care in asking for clarification, commenting, and answering.
      Check out our Code of Conduct.











      share|cite|improve this question









      New contributor




      João Simões is a new contributor to this site. Take care in asking for clarification, commenting, and answering.
      Check out our Code of Conduct.









      share|cite|improve this question




      share|cite|improve this question








      edited 2 hours ago









      Robert Howard

      1,417622




      1,417622






      New contributor




      João Simões is a new contributor to this site. Take care in asking for clarification, commenting, and answering.
      Check out our Code of Conduct.









      asked 2 hours ago









      João Simões

      83




      83




      New contributor




      João Simões is a new contributor to this site. Take care in asking for clarification, commenting, and answering.
      Check out our Code of Conduct.





      New contributor





      João Simões is a new contributor to this site. Take care in asking for clarification, commenting, and answering.
      Check out our Code of Conduct.






      João Simões is a new contributor to this site. Take care in asking for clarification, commenting, and answering.
      Check out our Code of Conduct.




















          2 Answers
          2






          active

          oldest

          votes

















          up vote
          2
          down vote



          accepted










          The quotient blows up near the line $x=y$ no matter how close to $(0,0)$ you get, so the limit does not exist.






          share|cite|improve this answer






















          • Small point: It's not a polynomial.
            – zhw.
            1 hour ago










          • @zhw: Right, fixed.
            – Henning Makholm
            24 mins ago

















          up vote
          3
          down vote













          Write
          beginalign
          dfracx^2+y^2+5xyx-y &= x-y +dfrac7xyx-y \
          &=x-y+yleft(dfrac7x-7y+7yx-yright) \
          &=x-y+7y+dfrac7y^2x-y=x+6y+dfrac7y^2x-y.
          endalign

          To see that the limit fails to exist, consider $2$ paths: the first one is $(x,y) = (2t,t)$,and the limit is $0$. And the second path is $(x,y) = (t^2+t,t)$, and the limit is $7$. Thus you have $2$ different values of the limit, hence it does not exist.






          share|cite|improve this answer






















          • You might want to use beginalign and endalign for readability. $(+1)$ by the way.
            – Mattos
            1 hour ago











          Your Answer





          StackExchange.ifUsing("editor", function ()
          return StackExchange.using("mathjaxEditing", function ()
          StackExchange.MarkdownEditor.creationCallbacks.add(function (editor, postfix)
          StackExchange.mathjaxEditing.prepareWmdForMathJax(editor, postfix, [["$", "$"], ["\\(","\\)"]]);
          );
          );
          , "mathjax-editing");

          StackExchange.ready(function()
          var channelOptions =
          tags: "".split(" "),
          id: "69"
          ;
          initTagRenderer("".split(" "), "".split(" "), channelOptions);

          StackExchange.using("externalEditor", function()
          // Have to fire editor after snippets, if snippets enabled
          if (StackExchange.settings.snippets.snippetsEnabled)
          StackExchange.using("snippets", function()
          createEditor();
          );

          else
          createEditor();

          );

          function createEditor()
          StackExchange.prepareEditor(
          heartbeatType: 'answer',
          convertImagesToLinks: true,
          noModals: true,
          showLowRepImageUploadWarning: true,
          reputationToPostImages: 10,
          bindNavPrevention: true,
          postfix: "",
          imageUploader:
          brandingHtml: "Powered by u003ca class="icon-imgur-white" href="https://imgur.com/"u003eu003c/au003e",
          contentPolicyHtml: "User contributions licensed under u003ca href="https://creativecommons.org/licenses/by-sa/3.0/"u003ecc by-sa 3.0 with attribution requiredu003c/au003e u003ca href="https://stackoverflow.com/legal/content-policy"u003e(content policy)u003c/au003e",
          allowUrls: true
          ,
          noCode: true, onDemand: true,
          discardSelector: ".discard-answer"
          ,immediatelyShowMarkdownHelp:true
          );



          );






          João Simões is a new contributor. Be nice, and check out our Code of Conduct.









           

          draft saved


          draft discarded


















          StackExchange.ready(
          function ()
          StackExchange.openid.initPostLogin('.new-post-login', 'https%3a%2f%2fmath.stackexchange.com%2fquestions%2f2982435%2fthis-polynomial-limit-doesnt-exist-but-i-cant-seem-to-prove-it%23new-answer', 'question_page');

          );

          Post as a guest






























          2 Answers
          2






          active

          oldest

          votes








          2 Answers
          2






          active

          oldest

          votes









          active

          oldest

          votes






          active

          oldest

          votes








          up vote
          2
          down vote



          accepted










          The quotient blows up near the line $x=y$ no matter how close to $(0,0)$ you get, so the limit does not exist.






          share|cite|improve this answer






















          • Small point: It's not a polynomial.
            – zhw.
            1 hour ago










          • @zhw: Right, fixed.
            – Henning Makholm
            24 mins ago














          up vote
          2
          down vote



          accepted










          The quotient blows up near the line $x=y$ no matter how close to $(0,0)$ you get, so the limit does not exist.






          share|cite|improve this answer






















          • Small point: It's not a polynomial.
            – zhw.
            1 hour ago










          • @zhw: Right, fixed.
            – Henning Makholm
            24 mins ago












          up vote
          2
          down vote



          accepted







          up vote
          2
          down vote



          accepted






          The quotient blows up near the line $x=y$ no matter how close to $(0,0)$ you get, so the limit does not exist.






          share|cite|improve this answer














          The quotient blows up near the line $x=y$ no matter how close to $(0,0)$ you get, so the limit does not exist.







          share|cite|improve this answer














          share|cite|improve this answer



          share|cite|improve this answer








          edited 24 mins ago

























          answered 2 hours ago









          Henning Makholm

          234k16299531




          234k16299531











          • Small point: It's not a polynomial.
            – zhw.
            1 hour ago










          • @zhw: Right, fixed.
            – Henning Makholm
            24 mins ago
















          • Small point: It's not a polynomial.
            – zhw.
            1 hour ago










          • @zhw: Right, fixed.
            – Henning Makholm
            24 mins ago















          Small point: It's not a polynomial.
          – zhw.
          1 hour ago




          Small point: It's not a polynomial.
          – zhw.
          1 hour ago












          @zhw: Right, fixed.
          – Henning Makholm
          24 mins ago




          @zhw: Right, fixed.
          – Henning Makholm
          24 mins ago










          up vote
          3
          down vote













          Write
          beginalign
          dfracx^2+y^2+5xyx-y &= x-y +dfrac7xyx-y \
          &=x-y+yleft(dfrac7x-7y+7yx-yright) \
          &=x-y+7y+dfrac7y^2x-y=x+6y+dfrac7y^2x-y.
          endalign

          To see that the limit fails to exist, consider $2$ paths: the first one is $(x,y) = (2t,t)$,and the limit is $0$. And the second path is $(x,y) = (t^2+t,t)$, and the limit is $7$. Thus you have $2$ different values of the limit, hence it does not exist.






          share|cite|improve this answer






















          • You might want to use beginalign and endalign for readability. $(+1)$ by the way.
            – Mattos
            1 hour ago















          up vote
          3
          down vote













          Write
          beginalign
          dfracx^2+y^2+5xyx-y &= x-y +dfrac7xyx-y \
          &=x-y+yleft(dfrac7x-7y+7yx-yright) \
          &=x-y+7y+dfrac7y^2x-y=x+6y+dfrac7y^2x-y.
          endalign

          To see that the limit fails to exist, consider $2$ paths: the first one is $(x,y) = (2t,t)$,and the limit is $0$. And the second path is $(x,y) = (t^2+t,t)$, and the limit is $7$. Thus you have $2$ different values of the limit, hence it does not exist.






          share|cite|improve this answer






















          • You might want to use beginalign and endalign for readability. $(+1)$ by the way.
            – Mattos
            1 hour ago













          up vote
          3
          down vote










          up vote
          3
          down vote









          Write
          beginalign
          dfracx^2+y^2+5xyx-y &= x-y +dfrac7xyx-y \
          &=x-y+yleft(dfrac7x-7y+7yx-yright) \
          &=x-y+7y+dfrac7y^2x-y=x+6y+dfrac7y^2x-y.
          endalign

          To see that the limit fails to exist, consider $2$ paths: the first one is $(x,y) = (2t,t)$,and the limit is $0$. And the second path is $(x,y) = (t^2+t,t)$, and the limit is $7$. Thus you have $2$ different values of the limit, hence it does not exist.






          share|cite|improve this answer














          Write
          beginalign
          dfracx^2+y^2+5xyx-y &= x-y +dfrac7xyx-y \
          &=x-y+yleft(dfrac7x-7y+7yx-yright) \
          &=x-y+7y+dfrac7y^2x-y=x+6y+dfrac7y^2x-y.
          endalign

          To see that the limit fails to exist, consider $2$ paths: the first one is $(x,y) = (2t,t)$,and the limit is $0$. And the second path is $(x,y) = (t^2+t,t)$, and the limit is $7$. Thus you have $2$ different values of the limit, hence it does not exist.







          share|cite|improve this answer














          share|cite|improve this answer



          share|cite|improve this answer








          edited 1 hour ago









          Rócherz

          2,2962518




          2,2962518










          answered 1 hour ago









          DeepSea

          69.9k54386




          69.9k54386











          • You might want to use beginalign and endalign for readability. $(+1)$ by the way.
            – Mattos
            1 hour ago

















          • You might want to use beginalign and endalign for readability. $(+1)$ by the way.
            – Mattos
            1 hour ago
















          You might want to use beginalign and endalign for readability. $(+1)$ by the way.
          – Mattos
          1 hour ago





          You might want to use beginalign and endalign for readability. $(+1)$ by the way.
          – Mattos
          1 hour ago











          João Simões is a new contributor. Be nice, and check out our Code of Conduct.









           

          draft saved


          draft discarded


















          João Simões is a new contributor. Be nice, and check out our Code of Conduct.












          João Simões is a new contributor. Be nice, and check out our Code of Conduct.











          João Simões is a new contributor. Be nice, and check out our Code of Conduct.













           


          draft saved


          draft discarded














          StackExchange.ready(
          function ()
          StackExchange.openid.initPostLogin('.new-post-login', 'https%3a%2f%2fmath.stackexchange.com%2fquestions%2f2982435%2fthis-polynomial-limit-doesnt-exist-but-i-cant-seem-to-prove-it%23new-answer', 'question_page');

          );

          Post as a guest













































































          Comments

          Popular posts from this blog

          Long meetings (6-7 hours a day): Being “babysat” by supervisor

          Is the Concept of Multiple Fantasy Races Scientifically Flawed? [closed]

          Confectionery